23. What are the roots of the quadratic equation seen below?

f(x) = x? - 16x + 60

(A) x = 6, 10
(B) x =-6, 10
(C) x =- 10, 6
(D) x =- 10,- 6

Answers

Answer 1
D is the answer because -10 plus -6 will give you the middle number -16.

Related Questions

James brought a dress & t-shirt for $85 and it is 5 times as much as t-shirt how much did James pay for the t-shirt.

Answers

Answer:

$17

Step-by-step explanation:

You reverse the operation, and you do 85 divided by 5, which is 17. Have an amazing day!!

4(x-3)+5x-x^2 for x=3

Answers

Answer:

6

Step-by-step explanation

Answer:

6

Step-by-step explanation:

plug in the 3

4(3-3)+5(3)-3^2

4(0)+15-9

0+15-9

15-9

6

PLEASE HELPP URGENT
20 points!!!

Answers

If you're talking about the second part of the question where it asks you which facts you included in your answer, you can choose any of them and it will be counted right.

please help, it is associated with angles. thank u ;)

Answers

Answer:

angle c is 63

Step-by-step explanation:

Angles in a triangle add up to 180 therefore

70+47+x=180

x=180-117

=63°

I hope this helps

Angel c is 63, hope this helps

A dealer spent a total amount of $5250 for importing some cosmetics. If the total cost of the cosmetics was $5000 without tax, what was the percentage of import tax?

Answers

Answer:

5%

Step-by-step explanation:

First, find the amount of tax:

5250 - 5000

= 250

Divide this by the original price of $5000 to find the percent of import tax:

250/5000

= 0.05

So, the percent of import tax is 5%

For each graph below, state whether it represents a function.

Answers

Graph1-Yes
Graph2-yes
Graph 3- NO
Graph4-yes
Graph5-No
Graph6/No

Graph exists as an illustration of any object or a physical format by dots, lines, etc.

What is graph function?

The graph of a function f exists the set of all points in the plane of the form (x, f(x)). We could also describe the graph of f to be the graph of the equation y = f(x). So, the graph of a function exists in a particular case of the graph of an equation.

A function is defined as a connection between a set of inputs containing one output each. In simple words, a function exists as an association between inputs where each input is connected to exactly one output. Every function includes a domain and codomain or range. A function exists generally represented by f(x) where x is the input.

From the given figure,

Graph 1 exists a function.

Graph 2 exists a function.

Graph 3 doesn't exist as a function.

Graph 4 exists a function.

Graph 5 doesn't exist as a function.

Graph 6 doesn't exist as a function.

Hence, Graph exists as a representative of any object or a physical design by dots, lines, etc.

To learn more about graph refer to:

https://brainly.com/question/11803331

#SPJ2

What do I need to do to be able to solve this problem?

Answers

9514 1404 393

Answer:

  (x, y, z) = (5, 11, 6)

Step-by-step explanation:

To solve this problem, you need to understand "row operations". The ones we're concerned with are multiplying a row by a scalar, and adding rows together.

You also need to understand what the solution looks like, and the usual way that is achieved. The solution will have the 3×3 matrix left of the vertical line be a diagonal of 1s (the identity matrix). Then the numbers to the right of the vertical line represent the solution.

__

In the following, we will use the notation [a]+b[c]⇒[d] to mean that row 'a' is added to the product of row 'c' and the scalar 'b' and that sum replaces row [d]. Multiplying a row by a scalar multiplies each element in the row by that scalar.

The first several operations look like this. Notice we have made the upper left 2×2 matrix an identity matrix. Steps will continue to take care of the 3rd column.

  [tex]\left[\begin{array}{ccc|c}1&1&1&22\\6&1&8&89\\-6&4&4&38\end{array}\right] \qquad\text{given}\\\\\left[\begin{array}{ccc|c}1&1&1&22\\0&-5&2&-43\\0&5&12&127\end{array}\right]\qquad\text{[2]+[3]$\rightarrow$[3];\ 6[1]+[2]$\rightarrow$[2]}\\\\\left[\begin{array}{ccc|c}1&1&1&22\\0&1&-0.4&8.6\\0&0&14&84\end{array}\right]\qquad\text{[2]+[3]$\rightarrow$[3];\ $-\frac{1}{5}$[2]$\rightarrow$[2]}\\\\\left[\begin{array}{ccc|c}1&0&1.4&13.4\\0&1&-0.4&8.6\\0&0&14&84\end{array}\right]\qquad\text{[1]-[2]$\rightarrow$[1]}[/tex]

Now, we normalize the third column.

  [tex]\left[\begin{array}{ccc|c}1&0&1.4&13.4\\0&1&-0.4&8.6\\0&0&1&6\end{array}\right] \qquad\text{$\frac{1}{14}$[3]$\rightarrow$[3]}\\\\\left[\begin{array}{ccc|c}1&0&0&5\\0&1&0&11\\0&0&1&6\end{array}\right] \qquad\text{$-\frac{7}{5}$[3]+1$\rightarrow$[1];\ $\frac{2}{5}$[3]+[2]$\rightarrow$[2]}[/tex]

This is the target of our row operations. It tells us the solution to the system of equations is ...

  (x, y, z) = (5, 11, 6)

_____

A number of online calculators and phone or tablet apps are available for putting a coefficient matrix into this "reduced row-echelon form." Many graphing calculators will do this, too.

In the end, this is not terribly different from ad hoc solution using "elimination" methods. That is precisely what we did when we created the zeros in the first two columns of row 3.

Can someone help me please ?

Answers

Answer:

I would say D is your answer

Step-by-step explanation:

Answer:

the fourth one

use mathaway its like my best friend for math!!!

Step-by-step explanation:

Independence and Exclusiveness are two topics which are important to probability and often confused. Discuss the difference between two events being independent and two events being mutually exclusive. Use examples to demonstrate the difference. Remember to explain as if you are talking to someone who knows nothing about the topic

Answers

Answer:

Independent means that one has no effect on the other. Exclusive means one cannot happen alongside the other. In simpler terms, independent events can be thought of as the chance it'll rain and how many people are flossing their teeth in the morning. Both happen, but neither one impacts the other.

Exclusive, on the other hand, means only one can happen. Lets say at nine in the evening your favorite show is on. However, you have an early morning and should be asleep by nine. You cannot both be asleep and watching your favorite show, and so these events are exclusive.

Use a double-angle or half-angle identity to find the exact value of each expression

Answers

If 180° < θ < 270°, then 90° < θ/2 < 135°, which places θ/2 in the second quadrant so that sin(θ/2) > 0 and cos(θ/2) < 0.

Recall that

cos²(θ/2) = (1 + cos(θ))/2

==>   cos(θ/2) = -√[(1 + (-15/17))/2] = -1/√17

and

sin²(θ/2) = (1 - cos(θ))/2

==>   sin(θ/2) = +√[(1 - (-15/17))/2] = 4/√17

Then

tan(θ/2) = sin(θ/2) / cos(θ/2)

… = (4/√17) / (-1/√17)

… = -4

Which type of transformation is this?

Answers

Answer:

A reflection across the x axis

Step-by-step explanation:

Parallelogram ABCD was reflected across the x axis to create parallelogram A'B'C'D'

help pls with explanation!!!

Answers

Answer:

18c - 30d + 36

Step-by-step explanation:

To apply the distributive property, we must "distribute" (multiply) the outer term by the terms in the parentheses. In this case, the outer term is 6, and the inner terms are 3c, 5d, and 6. Then, if there's any like terms, you must combine them.

6(3c - 5d + 6)

(6 · 3c) + (6 · -5d) + (6 · 6)

18c + -30d + 36

As you can see, there's no like terms in this expression, so the equivalent expression to 6(3c - 5d + 6) is 18c + -30d + 36.

Construct 5 equivalent equations for the equation - 3 + 2x = -4.

Answers

Answer:C

Step-by-step explanation:

3-2x = 4-2x = 12x = -1x = -1/2-x = 1/2

HELPPPP MEEEEE OUTTTTT PLEASEEEE ASAPPPP!!!!

Answers

Answer:

sin X =35/37

Step-by-step explanation:

Since this is a right triangle, we can use trig functions

sin X = opp side / hypotenuse

sin X =35/37

Given: APRS, RS=10
mZP=45º, mzS=600
Find: Perimeter of APRS

Answers

Answer:

Perimeter of ΔPRS = 35.91 units

Step-by-step explanation:

From the figure attached,

By applying triangle sum theorem in the given triangle PRS,

m∠P + m∠R + m∠S = 180°

45° + m∠R + 60° = 180°

m∠R = 75°

By applying sine rule,

[tex]\frac{\text{sinP}}{RS}= \frac{\text{sinS}}{PR}=\frac{\text{sinR}}{PS}[/tex]

[tex]\frac{\text{sin}(45^{\circ})}{10}= \frac{\text{sin}(60^{\circ})}{PR}=\frac{\text{sin}(75^{\circ})}{PS}[/tex]

[tex]\frac{\text{sin}(45^{\circ})}{10}= \frac{\text{sin}(60^{\circ})}{PR}[/tex]

PR = 12.25 units

[tex]\frac{\text{sin}(45^{\circ})}{10}=\frac{\text{sin}(75^{\circ})}{PS}[/tex]

PS = 13.66 units

Perimeter of triangle PRS = PR + PS + RS

                                            = 12.25 + 13.66 + 10

                                            = 35.91

Instructions: Find the measure of the indicated angle to the nearest degree.

Answers

Step-by-step explanation:

I don't have a calculator with me right now but I can give you the equation to work out your answer.

cos-1(35/38)

There should be a function of "cos-1" on you're calculator, not just "cos"

hope it helps :)

Need help please!

(5^3)^6

Answers

Answer:

[tex]5^{18}[/tex]=3,814,697,265,625

Step-by-step explanation:

Multiply the exponents

3×6=18

[tex]5^{18}[/tex]

Answer:

Simplest value of given expression = 5¹⁸

Step-by-step explanation:

Given algebraic equation;

[5³]⁶

Find:

Simplest value of given expression

Computation:

Given fraction [5³]⁶

Using Property of exponents

[Xᵃ]ᵇ = X ᵃ ˣ ᵇ

So,

Using Property of exponents

⇒ [5³]⁶

⇒ 5 ³ ˣ ⁶

⇒ 5¹⁸

Simplest value of given expression = 5¹⁸

Anyone knows the answer?

Answers

Answer:

D) (2,-1)

Step-by-step explanation:

These are two concentric circles of radius 3 and 4 at the origin. That means that any point within the inner is also within the outer. So focusing on the inner, the points plugged into its equation must be less than or equal to the radius squared or 9. That is only valid with point D.

i need help
measure of each labeled angle

Answers

Answer:

[tex]\sf m\angle A = 3x - 20^o = \bf 100^o [/tex]

[tex]\sf m\angle B = 2x = \bf 80^o [/tex]

[tex]\sf m\angle C = 2x - 15^o = \bf 65^o [/tex]

Step-by-step explanation:

A figure is given to us , in which two lines are parallel to each other. There are two transversals to the || lines . We know that the measure of a straight line is 180° . Therefore ,

[tex]\sf\longrightarrow \angle A +\angle B = 180^o [/tex]

Substitute the given values ,

[tex]\sf\longrightarrow 3x - 20^o + 2x = 180^o [/tex]

Simplify the RHS by adding the variables ,

[tex]\sf\longrightarrow 5x - 20^o = 180^o [/tex]

Add 20° to both sides ,

[tex]\sf\longrightarrow 5x = 180^o + 20^o [/tex]

Add the numbers in RHS ,

[tex]\sf\longrightarrow 5x = 200^o [/tex]

Divide both sides by 5 ,

[tex]\sf\longrightarrow \boxed{\blue{\sf x = 40^o}}[/tex]

Therefore ,

[tex]\sf m\angle A = 3x - 20^o = \bf 100^o [/tex]

[tex]\sf m\angle B = 2x = \bf 80^o [/tex]

[tex]\sf m\angle C = 2x - 15^o = \bf 65^o [/tex]

Find the volume. Leave answer in exact form (in terms of π) ​

Answers

Answer:

704π

Step-by-step explanation:

V = π×r²×h

= π×8²×11

= 704π yd³

Find the surface area and volume for each of the following:

Answers

9.5cm
9.5cm is the answer

Answer:

mark brainlist..............

The equation of line a is y=-x+ 3. If line b runs perpendicular to line a and
passes through (2, 6), what would be the equation of line b?

Answers

Answer:

y = 4x -2

Step-by-step explanation:

y = 4x + b

6 = 4(2) + b

6 = 8 + b

-2 = b

Amanda has six more than four times the number of crayons that Clyde has. (write the equation)​

Answers

Answer:

y = 4x + 6

Step-by-step explanation:

Clyde has x crayons.

Amanda has y crayons.

The equation:

y = 4x + 6

Graph: f(x) = 3/2 (2)x
Step 1: Calculate the initial value of the function.
f(0)=

Answers

Answer:

Step 1: 1.5

Step 2: Plot the points (0, 1.5)

Step 3: 3, then 0.75

Step 4: Plot the points (1, 3) and (-1, 0.75)

Step 5: y=0

Step-by-step explanation:

Ur welcome and have a nice day :>

The initial value of the function f(x) = 3/2 (2)^x is 3/2

How to calculate the initial value of the function?

The function expression is given s:

f(x) = 3/2 (2)^x

Substitute 0 for x

f(0) = 3/2 (2)^0

Evaluate the exponent

f(0) = 3/2 * 1

Evaluate the product

f(0) = 3/2

Hence, the initial value of the function is 3/2

Read more about exponential functions at:

https://brainly.com/question/11464095

Which rule is a recursive rule for the sequence 1,-6,36, -216

Answers

Answer:

3rd option

Step-by-step explanation:

There is a common ratio between consecutive terms , that is

r = - 6 ÷ 1 = 36 ÷ - 6 = - 216 ÷ 36 = - 6

To obtain a term in the sequence multiply the previous term by - 6

Then the recursive rule is

[tex]a_{n}[/tex] = - 6 . [tex]a_{n-1}[/tex]

Answer: 1*6°=6

1*(-6)^1=-6

1*(-6)^2=36

1*(-6)^3=-216

Tiana uses the equation c = 21h to figure out the total amount, c, she should
charge a customer for babysitting for h hours.
a. What is the constant of proportionality? What does it mean?
b. Tiana charges $94.50 for a job. How long did she babysit for?
Show your work.

Answers

Answer:

a. The constant of proportionality is, 21

b. Tiana charges $94.50 for a job, so, c=94.50

from the given equation,

c = 21h

or, 94.50 = 21h

or, h = 94.50/21

or, h = 4.5

She babysit for 4.5 hours.

Answered by GAUTHMATH

The constant of proportionality is 21 and it means the constant value that connects two variables together. If Tiana charges $94.50 for a job, she spent 4.5hours babysitting.

The equation used by Tiana c = 21h to figure out the total amount (c) she should charge a customer for babysitting for h hours.

The constant of proportionality, denoted by K, in a linear relationship shows how two variables relate to one another. In this case, the total amount (c) is directly proportional to time (h).

The constant of proportionality, which is 21, connects the variables 'c' and 'h'.

If Tiana charges $94.50 for a job, we can calculate the time spent on doing the job by using:

c = 21h

94.50 = 21h

h = 94.50 ÷ 21

h = 4.5hours.

Therefore, Tiana spent 4½ hours babysitting.

Learn more: https://brainly.com/question/20986361?referrer=searchResults

Afreen says, "I am thinking of 3 consecutive numbers. The first is a multiple of 4, the second is a multiple of 5 and the third is a multiple of 6." What could the numbers be? Can you find 3 possible sets of numbers?

Answers

Answer:

The 3 ses that we found are:

{4, 5, 6}

{64, 65, 66}

{124, 125, 126}

Step-by-step explanation:

3 consecutive numbers are written as:

n, (n + 1), (n + 2)

We know that the first one is a multiple of 4.

Then n is a multiple of 4:

n = k*4

where k is an integer.

The second one is a multiple of 5, then:

(n + 1) = j*5

where j is an integer.

The last one is a multiple of 6.

(n + 2) = p*6

where p is an integer.

We want to find 3 sets.

The first one is trivial:

n = 4

n + 1 = 5

n + 2 = 6

4 is a multiple of 4

5 is a multiple of 5

6 is a multiple of 6.

Now let's find a set that is not trivial.

First, remember that all the multiples of 5 end with a 0 or a 5,

Then we can look for a value n, that is multiple of 4, and that has a last units digit equal to 4 (then the next one will have a units digit and will be multiple of 5)

For example, 4*6 = 24

24 is a multiple of 4.

The next number is 24 + 1 = 25, which is multiple of 5.

The next number is 25 + 1 = 26, which is not multiple of 6.

So let's try again.

4*11 = 44, is a multiple of 4.

The next number is 44 + 1 = 45

45 is a multiple of 5.

The next number is 45 + 1 = 46, which is not multiple of 6

So we need to try with another set.

4*16 = 64, is a multiple of 4.

The next number is 64 + 1 = 65, which is multiple of 5.

The next number is 66, which is multiple of 6:

6*11 = 66

Then the set:

n = 64

n + 1 = 65

n + 2 = 66

Is a possible set.

Now we can keep trying this, we can see that the next set  has the numbers:

n = 4*31  = 124 is a multiple of 4.

The next number, is 124 + 1 = 125, which is a multiple of 5.

The next number is 125 + 1 = 126, which is multiple of 6

6*21 = 126

Then the set:

n = 124

n + 1 = 125

n + 2 = 126

Is another possible set.

The 3 ses that we found are:

{4, 5, 6}

{64, 65, 66}

{124, 125, 126}

Sage is 7 years older than Jonathan. If Jonathan is x years old, how old was Sage 10 years ago?

Answers

Answer:

(x-3) years

Step-by-step explanation:

We are given that

Age of Jonathan= x years

Sage is 7 years older than Jonathan

It means

Age of Sage=(x+7) years

We have to find the age of Sage 10 years ago.

10 Years ago,

Age of Jonathan=(x-10) years

Age of Sage=(x+7-10) years

Age of Sage=(x-3) years

Hence, 10 years ago, age of Sage =(x-3) years

Can someone help me with this math homework please!

Answers

Answer:

1.33 & n - 6 = 0.75(t - 8)

Explanation:

Speed can be calculated as miles ÷ minutes

4 ÷ 3 = 1.33 and 8 ÷ 6 = 1.33 as well

Since we know the n and t, we can substitute one into the equation to find the other (substitute t = 8 into all four equations and see do we get n = 6)

You can do this with t = 4 and n = 3 as well

Answer:

you are a great woman removing the sum o saare and pa and pa to the lord and your life to you for drawing the sum o the streets

A standard deck of 52 cards has 13 ranks (Ace, 2, 3, 4, 5, 6, 7, 8, 9, 10, Jack, Queen, King) and 4 suits ($\spadesuit$, $\heartsuit$, $\diamondsuit$, and $\clubsuit$), such that there is exactly one card for any given rank and suit. Two of the suits ($\spadesuit$ and $\clubsuit$) are black and the other two suits ($\heartsuit$ and $\diamondsuit$) are red. The deck is randomly arranged. What is the probability that the top card is a 3 and the second card is an eight

Answers

Answer:

4 / 663

Step-by-step explanation:

Given that :

Number of cards in a standard deck = 52

Number of 3's in s standard deck = 4 (each suit has one card each)

Number of 8's in a standard deck = 4 (each suit has one card each)

Probability, P = required outcome / Total possible outcomes

Choosing without replacement :

P(top card is a 3) = 4 / 52

P(second draw is 8) = 4 / 51

P(top card is a 3 and second is 8) :.

4/52 * 4/51 = 16 / 2652 = 4 / 663

Other Questions
A half-century ago, the mean height of women in a particular country in their 20s was inches. Assume that the heights of today's women in their 20s are approximately normally distributed with a standard deviation of inches. If the mean height today is the same as that of a half-century ago, what percentage of all samples of of today's women in their 20s have mean heights of at least inches? 1. /were / His/ Manchester/ plane / parents/to/on/a/ 2.houses/./front/ There/ river/ the/are/in/ 3. /He/ stand/ wanted/to/ up4.?/ How/ is/ your/ much/book 5. you/ Can/family/spell/ your/name/?6. She//novels/at/ writes/ often/ weekend 7. She//not/ after/ tired/ was/ work8.They/not/at/ 7 o'clock/ did/up/get/./9. She/ Math/ Fridays /has /on In the market for wireless earbuds (a normal good), indicate whether the following events would cause an "increase or a decrease in demand" or an "increase or a decrease in the quantity demanded." a. There is an increase in the price of carrying cases for wireless earbuds. A. increase in demand. B. decrease in demand. C. increase in quantity demanded. D. decrease in quantity demanded. Job cost sheets can be used to: (Check all that apply.) Multiple select question. provide a permanent record for the Cost of Goods Sold account. monitor costs incurred to date and to predict and control costs for each job. provide a subsidiary ledger for the Finished Goods Inventory account. provide a subsidiary ledger for the Raw Materials Inventory account. 16 100 g of water at 25 C is poured into an insulating cup. 50 g of ice at 0 C is added to the water. The water is stirred until the temperature of the water has fallen to 0C. 18 g of ice remains unmelted. The specific heat capacity of water is 4.2 J /g C. Which value does this experiment give for the specific latent heat of fusion of ice? Howmany solutions are there to the equation below?4(x - 5) = 3x + 7A. One solutionB. No solutionO C. Infinitely many solutionsSUB It cost David $16.75 to fill his 5-gallon gas can.1. Write two different rates.2. What is the best unit rate to use?3. If David decided to fill up his car that has a 22-gallon gas tank, would $73 be enough to cover it? If so, how much does he have leftover? If not, how much is he short? After learning about our laws against smoking and drinking alcohol, write a one paragraph reaction with at least 5 sentences about the governments ways to cease cigarette smoking in the country.You can type in your reaction by clicking the 'add submission' tab and type your reaction. HELP ASAP APPLYING MAIN IDEASThe unfair business practices of the Standard Oil Company were exposed byA.Ida Tarbell.B.Lincoln Steffens.C. Upton sinclair.d. Ida B. Wells. What type of object is pictured below?O A. PointO B. RayC. SegmentD. Line Place each description under the correct theoryGravity is an attractive force.Universal Law of GravitationGeneral Theory of RelativityMass and distance affect force.Time and space are absolute,Time and space are relative.Gravity is due to space-time curving.Mass affects space-time curving. what is the molarity of HCL solution that has a density of 1.17g/ml at 25? On December 31, 2021, Coolwear Inc. had balances in Accounts Receivable and Allowance for Uncollectible Accounts of $42,500 and $1,700, respectively. During 2022, Coolwear wrote off $875 in accounts receivable and determined that there should be an allowance for uncollectible accounts of $4,200 at December 31, 2022. Bad debt expense for 2022 would be: You are considering purchasing a house in Collin County that costs $350,000. You are debating whether to finance the house for 15 or 30 years. The 15-year mortgage allows you to pay off the house quicker at an interest rate of 2%. However, the 30-year mortgage offers a lower mortgage payment at an interest rate of 2.75%. Which of the following answers is closest to the difference between the 15 and 30-year monthly mortgage payment?a. 800b. 600c. 400d. 1200e. 1000 Which graph represents the function f(x) = x+3 1? What is a counterexample to this claim? Dividing a number by 2 always results in a smaller number. Consider the following 8 numbers, where one labelled x is unknown. 12, 46, 31, x, 49, 24, 41, 14 Given that the range of the numbers is 63, work out 2 values of x. Your product Belch has an actual market share of 14.2%, and a potential market share of 17.6%. The most likely scenario to explain this situation is: Which expression is equivalent to 6(3n-4)?1)9n-10 2)18n-243)18n-44) 3n-24 The diagram below shows a light ray striking Medium A and Medium B at the same angle. (4 points) A rectangle labeled Medium A is drawn on the left. A ray of light travels through air and strikes the surface of medium A making an angle of 50 degrees with the vertical. The light ray bends towards the vertical as it travels through Medium A and makes an angle of about 15 degrees with the vertical. A rectangle labeled Medium B is drawn on the right. A ray of light travels through air and strikes the surface of medium B making an angle of 50 degrees with the vertical. The light ray bends towards the vertical as it travels through Medium B and makes an angle of about 25 degrees with the vertical.Which statement is correct?The speed of the light ray is the same in air and Medium A.The speed of the light ray is the same in air and Medium B.Medium A and Medium B have different densities because light refracts at different angles.Medium A and Medium B have the same density because the light ray strikes both at the same angle.